[seqfan] On prime(n) consecutive integers

Vladimir Shevelev shevelev at bgu.ac.il
Mon Feb 18 09:54:39 CET 2013


Consider sequence: a(n) (n>=3) is the minimal m>=0 such that the divided on prime(n) sum of prime(n) consecutive integers beginning with m has form 2^k*p, where k>=0 and p is prime.
a(3)=0, since (0+1+2+3+4)/5=2=2^0*p, where p=2 (I submitted a close sequence A209287). 
Note that records 0,1,2,3,... we get in places 3,8,20,45,... Can anyone continue the sequence of record positions?

Regards,
Vladimir

 Shevelev Vladimir‎



More information about the SeqFan mailing list